Sunteți pe pagina 1din 13

(math013)[2008](f)final~2816^_10006.pdf downloaded by cijli from http://petergao.net/ustpastpaper/down.php?course=MATH013&id=5 at 2014-10-04 21:02:19. Academic use within HKUST only.

Math013 Calculus I
Final Exam Solution, Fall 08
1. (12 = 3 + 3 + 3 + 3 points) Here is a table of some values of three differentiable functions a, b, and
c. The derivatives a , b and c are assumed to be continuous.
x

a(x) a (x) b(x) b (x) c(x) c (x)

1
2

12

19

Answer the following: An answer of true means true in all situations An answer of false means there
is a case where the assertion is false.
(i) (3 points) The function c has an inverse.
True

False

Brief reason.

Solution
The function c might not be one-to-one, e.g., we could have c(1) = 0 = c(2).
(In fact, the function can not have an inverse. c(2) = 0 and c (2) = 1 imply that c would decrease
into negative values before taking c(5) = 3, hence by continuity, c(x0 ) = 0 = c(2) for some point
2 < x0 < 5; i.e., c is not one-to-one.)

(ii) (3 points) The derivative a takes on the value 1 in the interval 0 x 2.


True
Solution

False

Brief reason.

Note that

2.

a(2) a(0)
= 1. By the mean value theorem, a (x0 ) = 1 for some 0 < x0 <
20

Compute the following:


(iii) (3 points) g(2) and g (2), where g(x) = b(a(x))
Solution

g(2) = b(a(2)) = b(0) = 5,


g (2) = b (a(2))a (2) = b (0)a (2) = 1 0 = 0

(iv) (3 points) Suppose b is an increasing function, and h is the inverse function to b. Compute h(9)
and h (9).
Solution

h(9) = 2, since b(2) = 9;


1
1
h (9) =
= by the chain rule: b(h(x)) = x = b (h(x))h (x) = 1.
b (2)
2

(math013)[2008](f)final~2816^_10006.pdf downloaded by cijli from http://petergao.net/ustpastpaper/down.php?course=MATH013&id=5 at 2014-10-04 21:02:19. Academic use within HKUST only.

2
2. (10 = 5 + 5 points) A 6-ft-tall man is walking away from a 24-ft-tall lamppost at a rate of 3 ft/s.
(i) (5 points) At what rate is the length of his shadow changing when he is 16 ft away from the
lamppost?
Solution

Noting that

s
6
1
x
s
x
=
= , or =
, we have s = , and
s+x
24
4
6
24 6
3


ds
1 dx
1
=
= 3 = 1 (ft/s)


dt x=16 3 dt x=16 3
L

24 ft

6 ft
x

s
shadow

(ii) (5 points) At what rate is the distance between the lamp and the tip of his shadow changing when
he is 16 ft away from the lamppost?
Solution

Noting that L2 = (s + x)2 + 242 , we have

dL2
d(s + x)2 d242
=
+
dt
dt
dt

dL
ds dx 
2L
= 2(s + x)
+
dt
dt
dt
p

16
Putting in x = 16, s = , and L = (16/3) + 16)2 + 242 = 38 145, we have
3

8
dL
64
145
= (1 + 3)
3
dt x=16
3

dL
32
32

145 (ft/s)
=
=
dt x=16
145
145
( 2.6575 ft/s.)

(math013)[2008](f)final~2816^_10006.pdf downloaded by cijli from http://petergao.net/ustpastpaper/down.php?course=MATH013&id=5 at 2014-10-04 21:02:19. Academic use within HKUST only.

3
3. (10 = 5 + 5 points) Consider the parabola y = x2 .
(i) (5 points) Write down the equations of the tangent and normal lines to the parabola at the point
(b, b2 ).
Solution
y = 2x, hence:
slope of the tangent line at (b, b2 ) = 2b,
1
slope of the normal line at (b, b2 ) = , if b 6= 0.
2b

Tangent line equation is y = 2bx b2

Normal line equation is y =

x
1
+ + b2 if b 6= 0; otherwise x = 0
2b 2

(ii) (5 points) The figure shows a circle of radius 2 inscribed in the parabola. Determine the center of
the circle.
Solution
By symmetry, the center is located at (0, a).
If (b, b2 ) is one of the intersection points of the circle and the
parabola, we have
(
 1
2
(b2 a)2 + b2 = 4
4 +b =4

b2 a
1
a = b2 + 12
b0 = 2b

15
17
Thus b =
, and a = .
2
4

Center is located at (x, y) =

 17 
0,
4

(math013)[2008](f)final~2816^_10006.pdf downloaded by cijli from http://petergao.net/ustpastpaper/down.php?course=MATH013&id=5 at 2014-10-04 21:02:19. Academic use within HKUST only.

4
4. (9 = 3 + 6 points) Consider the equation ln x = 4 sin x, for 0 < x 3.
(i) (3 points) How many solutions to the equation are there in the given interval? Give brief reason
for your answer.
Solution
Note that ln x is increasing, and 4 sin x is oscillating between
4 and 4.
Also, ln 2 < 4 sin(1), and ln 3 = 2.24334 to five decimal
places.
A rough sketch shows that there should be three solutions to
ln x = 4 sin x.
(Or more precisely, apply the intermediate value theorem on
the intervals [/2, 3/2], [3/2, 5/2], [5/2, 3] respectively
to show that the graph of 4 sin x can hit the graph of ln x
exactly once in each interval.)

5 y
4

y = 4 sin x

y = ln x

2
1
/2

x
/2

3/2 2 5/2

2
3
4
5

(ii) (6 points) Starting with the point x = 2.5, use Newtons method to approximate the smallest
solution to an accuracy of 0.001. Retain five decimal places in your calculation. (Hint: A solution
to ln x = 4 sin x is the same as a solution to 0 = f (x) = ln x 4 sin x. )
Solution
Let f (x) = ln x 4 sin x, then f (x) = x1 4 cos x.
Note that x = 2.5 is in the interval [/2, 3/2] which contains the smallest solution.
The recursive formula of the Newtons method is
xn+1 = xn
x0 = 2.50000
x1 = 2.50000
x2 = 2.87459
x3 = 2.87446
x4 = 2.87446

f (xn )
ln xn 4 sin xn
= xn 1

f (xn )
xn 4 cos xn

ln 2.500004 sin 2.50000


1
4 cos 2.50000
2.50000

= 2.90992
=

smallest solution = 2.874

(math013)[2008](f)final~2816^_10006.pdf downloaded by cijli from http://petergao.net/ustpastpaper/down.php?course=MATH013&id=5 at 2014-10-04 21:02:19. Academic use within HKUST only.

5
5. (14 = 3 + 5 + 3 + 3 points) Three copies of the graph of a function y = f (x) is drawn below. For
a 0, set
Z a
F (a) =
f (t) dt .
0

(i) (3 points) On the three graphs, indicate an area which equals:


(1) F (4)
y
y = f(x)
3
2
1

-1

F (4) =

f (t)dt
0

-1

(2) F (4 + h)
Solution

For an h > 0,
y
y = f(x)
3
2
1

F (4+h)=

4+h

f (t)dt

-1

-1

4+h

(3) F (4 + h) F (4)
y
y = f(x)
3
2
1

F (4+h)F (4)=

4+h

f (t)dt

-1

2
-1

4+h

(math013)[2008](f)final~2816^_10006.pdf downloaded by cijli from http://petergao.net/ustpastpaper/down.php?course=MATH013&id=5 at 2014-10-04 21:02:19. Academic use within HKUST only.

6
(ii) Determine, with explanation, the limits:
(1) (5 points)
lim

h0

F (4 + h) F (4)

4+h 4

f (2) = 3

Solution
Applying the squeeze theorem (sandwich theorem) to the area inequalities for
small enough h > 0,

f (2)( 4 + h 4) F (4 + h) F (4) f ( 4 + h)( 4 + h 4)

F (4 + h) F (4)

lim f (2) lim


lim f ( 4 + h)
y
y = f(x)
h0
h0
h0
4+h 4
3
F (4 + h) F (4)
2

we have lim
= f (2) = 3
h0
4+h 4

f (2)

-1

2
-1

(2) (3 points)
lim

h0

Solution

4+h
h

)
4

1
4

4+h

4 + h 4)( 4 + h + 4)
1
1

lim
= lim
= lim
h0
h0
h0
4
h( 4 + h + 4)
4+h+ 4



dx1/2
1
1
Or the limit is
= x1/2
= .

dx x=4 2
4
x=4

4+h
h

f ( 4+h)

(3) (3 points)
lim

h0

Solution

F (4 + h) F (4)
h

3
4

4+h 4
F (4 + h) F (4)

h
4+h 4

4+h 4
F (4 + h) F (4)
1
3

= lim
lim
=3 =
h0
h0
h
4
4
4+h 4

F (4 + h) F (4)
lim
= lim
h0
h0
h

(math013)[2008](f)final~2816^_10006.pdf downloaded by cijli from http://petergao.net/ustpastpaper/down.php?course=MATH013&id=5 at 2014-10-04 21:02:19. Academic use within HKUST only.

7
6. (13 = 2 + 4 + 7 points)
(i) (2 points) Find the derivative of the function y = f (x) = x sin(x).
Solution

f (x) = 1 cos x.

(ii) (4 points) Use part (i) to explain why sin(x) x for all x 0.
Solution
f (x) is non-decreasing since f (x) = 1 cos x 0, for all x .
Hence for any x 0, x sin x 0 sin 0 = 0, or sin x x.
(Or use the mean value theorem.)
(iii) (7 = 3 + 2 + 2 points)
a2
(1) (3 points) By integrating over a suitable interval use part (ii) to explain why cos(a) 1
2
for all a 0.
Solution

sin x x for any x 0, and hence for any a 0,


Z a
Z a
sin xdx
xdx
0

cos x

cos a + 1

a2
2

i1
0

a2
2

b3
for all b 0.
6

cos a 1 a2 /2 for any a 0, and hence for any b 0,


Z b
Z b
a2 
1
cos ada
da
2
0
0
h
ib h
a3 ib
sin a a
6 0
0
3
b
sin b b
6

(3) (2 points) Repeat to explain why cos(c) 1


Solution

cos a 1

(2) (2 points) Repeat to explain why sin(b) b


Solution

h x2 i a

c2
c4
+
for all c 0.
2
24

Integrating sin b and b b6 over the interval [0, c] for any c 0, we have
Z c
Z c
b3 
sin bdb
b
db
6
0
0
h
i c h b2
b4 i c
cos b

2
24 0
0
2
c
c4
cos c + 1

2
24
c2
c4
+
cos c 1
2
24

(math013)[2008](f)final~2816^_10006.pdf downloaded by cijli from http://petergao.net/ustpastpaper/down.php?course=MATH013&id=5 at 2014-10-04 21:02:19. Academic use within HKUST only.

7. (6 points) Express

1+

2 + +

n n

interval, and then find


lim

as a Riemann sum of a suitable function over certain

1+

n+

Solution
lim

n+

1+

2 + +

n n

!
n

!
2 + + n

.
n n

1
=
n

1
1
+
n n

2
1
+ +
n
n

n
n

1
is the right-endpoint Riemann sum of the function f (x) = x = x 2 over the interval [0, 1] divided
into n subintervals of equal length, hence


! Z 1
1 + 2 + + n
2 3/2 1 2
1/2

lim
=
x dx = x =
n+
n n
3
3
0
0

(math013)[2008](f)final~2816^_10006.pdf downloaded by cijli from http://petergao.net/ustpastpaper/down.php?course=MATH013&id=5 at 2014-10-04 21:02:19. Academic use within HKUST only.

9
8. (18 = 2 + 7 + 2 + 7 points) Consider a circular sector of radius R and angle .
(i) (2 points) Determine the area A of the circular sector in terms of R and .

Solution
Area A

1 2
2R

(ii) (7 points) The two straight edges of the circular sector are attached to one another to form a conical
cup. Determine the volume V of this conical cup.
Solution
The base radius r of the conical cup is: 2r = R r = R
2
q

2 2
The height of the conical cup is: h = R2 r 2 = R2 R42
h=R

2
4 2

The volume of the conical cup is


1 R2 2
1
R
V = r 2 h =
3
3 4 2
R3 2
Volume V =
12

(iii) (2 points) Let x =


Solution

2
4 2

2 or R3 2 p 2
=
4 2
4 2
24 2

. Rewrite, A and V in terms of R and x.


2
Area A =
Volume V =

R2 x
1 3 2
R x 1 x2
3

(iv) (7 points) Suppose you wish to have the conical cup have volume 27 cm3 . Determine x so that
3
the area A is minimal. (Hint: The value of A will be minimal when the value of A 2 is minimal.)
Solution

V = 1 R3 x2 1 x2 = 27
3

A = R2 x

A3/2 = 3/2 R3 x3/2 =

81 3/2
81 3/2

=
,
x1/2 1 x2
x x3

(x > 0)

A3/2 achieves its minimum when p(x) = x x3 achieves its maximum. p(x) is concave down on the
interval x > 0 since p (x) = 6x < 0 there, hence its maximum is achieved at the critical number
1
1 3x2 = 0, i.e., at x = .
3
d 3/2
3/2
(Or by considering directly
A = 812 (x x3 )3/2 (1 3x2 ) = 0 .)
dx

(math013)[2008](f)final~2816^_10006.pdf downloaded by cijli from http://petergao.net/ustpastpaper/down.php?course=MATH013&id=5 at 2014-10-04 21:02:19. Academic use within HKUST only.

10
9. (10 = 2 + 2 + 6 points) Consider the real numbers R. Define a function f with domain R as follows:
Imagine wrapping the real axis around the equator of the earth. Write x w(x) for this function.
Then, compose the wrapping function w with the function which gives the temperature T . So,
f (x) = T (w(x)) is the function which at input x outputs the temperature of the equator point
w(x). You may assume the function f is continuous.
(i) (2 points) Let L be the distance around the earths equator. Explain why f (x + L) = f (x).
Solution
w(x) and w(x + L) are the same point on the equator, hence the same temperature
there: T (w(x + L)) = T (w(x)).

(ii) (2 points) Explain why the function f achieves a maximum on the open interval < x < +.
Solution
The continuous function f (x) must achieve a maximum on the closed interal [0, L],
which must be the maximum on < x < by the periodic property of f in (i).

(iii) (6 points) Consider the function with domain R defined as h(x) := f (x) f (x + L2 ). The interpretation of the function h is that it is the difference in temperatures at a point w(x) on the equator
and the diametrically opposite point on the equator. Choose which of the following theorems:
(a) Intermediate value theorem
(b) Mean value theorem
(c) Squeeze theorem
allows you to conclude that the function h is zero at some point. This means there is at least one
pair of diametrically opposite points on the equator which have the same temperature. For your
selection, you should state what the theorem asserts, and briefly explain how it allows you to make
the conclusion above.
Solution
The intermediate value theorem asserts that any real number between F (a) and F (b)
of a function F continuous on [a, b] must also be a function value F (c) for some a < c < b.
Note that h(x) = f (x) f (x + L2 ) is continuous everywhere since f is continuous. Consider any x0 ,
h(x0 + L/2) = f (x0 + L/2) f (x0 + L/2 + L/2) = f (x0 + L/2) f (x0 ) = h(x0 )
So, either h(x0 ) = 0, or h(x0 ) 6= 0. If h(x0 ) 6= 0, then 0 is a number between the two function values
h(x0 ) and h(x0 + L/2) with opposite signs, and the intermediate value theorem says that 0 = h(c)
for some c between x0 and x0 + L2 . In any case, h is zero at some point.

(math013)[2008](f)final~2816^_10006.pdf downloaded by cijli from http://petergao.net/ustpastpaper/down.php?course=MATH013&id=5 at 2014-10-04 21:02:19. Academic use within HKUST only.

11
10. (16 = 4 + 4 + 4 + 2 + 2 points) The graph of a function f is
y
3

y = f(x)

2
1

-1
-2

(i) (4 points) Set g(x) = f (x). Sketch the graph of g on the above graph of f .

(ii) Set h(x) =

f (t)dt.
0

(1) (4 points) On the copy of the graph below, sketch the graph of h.
y
3

y = f(x)

2
1

-1
-2

local max at x = 3, local min at x = 4.5


inflection points when x = 1, 4, 5, or 6

(2) (4 points) On the graph of h sketched above, indicate at which values x the local maximum and
minimum values of h occur. Be sure to label your points local maximum or local minimum.
Solution
Local maximum at x = 3, local minimum at x = 4.5.
(3) (2 points) Indicate any inflection points of h.
Solution
Inflection point occurs at x = 1, 4, 5, or 6.
(4) (2 points) Indicate intervals where h concave downward or upwards.
Solution
Concave upward: 0 < x < 1, 4 < x < 5, or 6 < x < 7
Concave downward: 1 < x < 4, or 5 < x < 6.

(math013)[2008](f)final~2816^_10006.pdf downloaded by cijli from http://petergao.net/ustpastpaper/down.php?course=MATH013&id=5 at 2014-10-04 21:02:19. Academic use within HKUST only.

12
11. (12 = 4 + 8 points) A bank normally gives deposits 2% interest compounded continuously. This
means if y is the amount of money in an account, then
dy
= 0.02 y , with t measured in years.
dt
1 dy
= 0.02. Find anti-derivatives for both sides and then find the
y dt
explicit expression for y(t) assuming y(0) = 100.

(i) (4 points) Rewrite this as

Solution
Z 
Z
1 dy 
dt = 0.02dt
y dt
ln |y| = 0.02t + C

Putting in y(0) = 100, we have C = ln 100, and


y(t) = e0.02t+ln 100 = 100e0.02t .

(ii) (8 points) As a promotion for new deposits, the bank gives interest according to the rate

0.08(1 t) + 0.02 for 0 t 1


dy
= h(t) y , where h(t) =

dt

0.02
for 1 < t
which the bank advertises as 10% initial interest.

(1) (2 points) Write the continuous anti-derivative H of h which satisfies the condition H(0) = 0
in the same manner as h is described.
Solution

For 0 t 1,
Z
Z
H(t) = [0.08(1 t) + 0.02]dt = (0.1 0.08t)dt = 0.1t 0.04t2 + C

Putting in H(0) = 0, we have C = 0, and H(t) = 0.1t 0.04t2 for 0 t 1. For t > 1,
Z
H(t) = 0.02dt = 0.02t + C .
To be continuous, we must have 0.02 + C = H(1) = 0.1 0.04, i.e., C = 0.04, and H(t) =
0.02t + 0.04 for t > 1.

H(t) =

0.1t 0.04t

0.02t + 0.04

for 0 t 1
for 1 < t

(math013)[2008](f)final~2816^_10006.pdf downloaded by cijli from http://petergao.net/ustpastpaper/down.php?course=MATH013&id=5 at 2014-10-04 21:02:19. Academic use within HKUST only.

13
(2) (5 points) Find the explicit piecewise expression for y(t) assuming y(0) = 100.
Solution

Either use H directly, or consider again

1 dy
= h(t). For 0 t 1,
y dt

ln |y| = H(t) + C = 0.1t 0.04t2 + C


2

and C = ln 100, i.e., y(t) = 100e0.1t0.04t .


We also have for t > 1,
ln |y| = H(t) + C = 0.02t + C .

Noting that y(1) = 100e0.06 , and ln y(1) = ln 100 + 0.06, we have C = ln 100 + 0.04.
(
2
100e0.1t0.04t , 0 t 1,
y(t) =
100e0.02t+0.04 , t > 1 .

(3) (1 point) After one year (t = 1), how much additional interest would the promotion plan earn
when compared with the (non promotion) first plan?
Solution
additional interest = (100e0.10.04 100) (100e0.02 100)
= 100(e0.06 e0.02 ) = 4.16 (to two decimal places.)

S-ar putea să vă placă și